The smart way to learn. The smart way to teach.

MRCOG PART 2 SBAs and EMQs

Course PAID
notes336
EMQ1502
SBA2115
Do you realy want to delete this discussion?
Forum >>

Essay 341

Posted by miss T.

(a)

Detailed history need to be taken trying to get reasons for previous failures, risk assessment for avaialable contraceptive options and her future reproductive intentions.

I will start from her menstrual history, LMP periods cyclicity, unexplained bleeding, associated symptoms like pain, pre-menstrual tension as it will be considered while choosing any method.

Details of previous contraceptives, duration of use, method use that she had followed and from where did she get to know about that method, any history of method failure or unprotected intercourse. It will help knowing that was she counselled appropriately about the use of particular method as it may be the place to focus this time. Was she compliant with the treatment which happen with every day use methods and LARC may be considered this time. And was the failure related to typical or perfect use.

Medical history and risk assessment for different methods including risk factors ot VTE (personal or family history of VTE) cardiovascular event or stroke (hypertension, uncontrolled diabetes, hyperlipidemia, smoking, h/o MI or stroke), unexplained vaginal bleeding, known to have or treated endometrial, cervical or breast cancer, risk factors of STI or on treatment, migraine or any medical illness. family history of inhereted cancers etc. Any treatment that she is on which is likely to reduce efficacy of hormonal contraceptives like liver enzyme inducing drugs.

Her future reproductive intentions, for how long does she intend to use contraception cause LARC may be considered and is she in a stable relatoinship as she may also need advise for prevention of STI.

Examination will include blood pressure and BMI (which we know already). Pelvis assessment is unnessaru unless there are symtoms suggestive of pelvic pathology or if IUCD is chosen for contraception.

(b)

There is no method with 100% success but maximum benefit can be achieved with perfect use.

The contraception hormonal options include combined pills, progestin only preparations (pill,s implant, injection and IUS).

She will be given evidence based information about all the methods and her decision will be respecetd. 

Given her BMI of 42, COCP is not an option for her since it is MEC 4 with unacceptable risks to her health because of associated increase in VTE and CVS event.

Progestin only methods (pills, implants and injectables) are not contraindicated in her case with this BMI given that she does not have current breast cancer but compliance with pills is an issue. Most POP should be taken at the same time each day and the delay if more than 3 hours (12 hours for cerazzete) can result in pregnancy and since she already has 3 unplanned pregnancies it would not be advised unless patient opt for it.

Progestin Implants, contain 68 mg of etonorgestril that is inserted for 3 years. It comes in category of LARC and for her it is a good option as it has added benefit of reducing dysmenorhoea and ? PMS but there are side effects like irregular vaginal bleedng in 10-20% of patients which is the main reason of discontinuation. It does not cause delay in the return of fertility.

Injectable: MPA  is given every 12 weeks and norehisterone enethate every 8 weeks. It is suitable contraception but is associated with a delay in return of fertility of upto 24 months and it is associated with decrease in BMD since she has another risk factor of morbid obesity this need to be explained in detail.

LNG-IUS is licensed for 5 years as contraceptive and is also good for patient who have compliance problem. It contain 52 mg levonorgestril and 20mcg is released per day. It has added benefit of reducing mean menstrual blood loss, dysmenorhea, reduce PID and ectopic when compared to Cu IUCD, known to reduce fibroid growth and given endometrial protection to unopposed estrogena activity. It does not cause any delay in return of fertilty. She will be told of small increase risk of infection in first 3 weeks of insertion, perforation 1:200, difficult insertion, irregular bleeding of 3-6 months. After one year of use it has amenorrhoea rate of 35%. It can not be used as emergency contraception.

There are EC preparations: levonorgestril 1.5 mg, sinle dose that need to be taken withing 72 hours of unprotected SI. Although not hormonal methos but in context of EC, Cu IUCD will be discusse with her which can be placed with 5 days of UPSI or suspected ovulation date. 

Written information leaflet will be provided.

(c)

The options are copper containing devices or hormonal intrauterine system. Both come in the category of LARC and there use of cost effective. The failure rate is .1% with copper device and .2% with LNG-IUS. Cu IUCD act by preventing fertilization and LNG-IUS by inhibiting implantation.

They can be inserted in the absence of contraindications unexplained vaginal bleeding, active STI, distorted cavity, endometrial or cervical cancer awaiting treatement, previous molar pregnancy with abnormal BHCG levels.

It can be fitted in first 5 days of menstrual cycle or any time if she is reasonable certain that she is not pregnant. Cu IUCD is immediately effective but with LNG if placed after first 5 days of cycle additional contraceptive is advised for 7 days.

The side effects are small risk of perforation, expulsion, 6 fold increase risk of STI in first 3 weeks of insertion.

She will be seen 3 weeks after insertion of IUCD to check its in place and no additional problms and after that no follow up is needed and she will be educated how to check for thread at home. If she want to remove the device it will be by the doctor and there is no delay in return of fertility.

It is meant for contraception and does not protect against STI, and if she is high risk (age 22, multiple partners, smoker etc) she will be advised to use barrier methods (eg condoms) for safer sex.

She will be informed of alternative methods as progestin implants, injections, sterlisation, partner sterlisation.

Written information leflet will be given.

Essay 341 by N Posted by Sherif N.
proper history taking is mandatory to detect any medical problem the patient is having. pt is obviously morbidly obese, it is important to know whether she is having Dibetes Mellitus, hypertension, cardiac problems, chest troubles, thrombophilic disorders, any previous DVT. also whether she is having regular periods or menorrhagia, intermenstrual bleeding, undiagnosed vaginal blleding, or vaginal discharge, any history of PID, if she has undergone any previous operation. also family history of breast cancer, cancer ovary, endometrium, history of thrombophilia, strokes, embolism, heart troubles, hypertension, diabetes mellitus general examination for her BP, heart, chest, sighns of DVT, local examination for size of uterus, cervical abnormalities, vaginal discharge, or bleeding, vaginal swab to be taken TVS to detect size of uterus, any uterine anomalies, endometrial thickness, any ovarian cysts or PCO, masses. lab test in the form of pregnancy test, FBC, blood sugar, U&E, liver functions, thrombophilia screen, urine analysis b) after thorough history, medical examination and labs, pt is having only high BMI, no other risk factors, she can be offered hormonal contraception in the form of: POPs, to be taken daily non stop, from 1st day of menses with no additional protection, otherwise she needs additional protection for 48hrs. 3rd generation COCP also can be offererd to be taken for 21days and 1week rest, starting from 1st day of menses with no additional protection, otherwise 48hrs protection is required too contraceptive patches may be offered to be put weekly, with one week rest injectables DMPA every 3months all these hormonal types, pt should be councelled and explained that there may be spotting and intermittent bleeding during the 1st 3months of the intake of hormornal contraception, and this will settle down with regular intake later, she may have to take mefenamic acid to control this bleeding, also she can add 2nd generation COCP to DMPA injection, or change to another type of hormone if bleeding persists for more than 3months (after consulting her physician), she should be informed also that this intermittent bleeding is unpredictable, and there is no evidence that one kind of hormone is causing more bleeding than the other, it depends upon pt's response to the knid of contraception also pt's wish should be taken in consideration (whether she like pills, injectables or patches)her compliance and her satisfaction c) pt could have an Intrauterine device (if not contraindicated, no pregnancy, no undiagnosed genital bleeding, no acute PID, or STIs) after taking vaginal swab and tt for infection if required. these types may be: copper T 380, can be felt for 8yrs, Gynefix, Nova-T 375, can be left for 5yrs, these are non hormonal forms, which may increase the amount and duration of menses, so patient should be counselled about that prior to insertion if she is having already heavy periods, so it is better to insert Mirena (levonorgestrel releasing IUD, which releases 20mcg/day for 5yrs), she should be counselled of the possibility of intermittent bleeding in the first 3months,this is the main cause of discontinuation and dis-satisfaction, so it should be thouroughly explained to the patient, but this bleeding will subside gradually, and oligomenorrhea or amenorrhea may occur...it may be needed to be changed before the 5years duration as the hormonal effect will decrease later in this morbidly obese lady (if she remains with this weight) it is important always that the pt knows the side effects, advantage and disadvantage of each method, and her wishes to be put into consideration so long that the method she chooses isn't contraindicated
Hodder Posted by Hodder C.
She is categorized under morbid obesity which puts her to the risk of development of VTE, MI,hypertension and endometial cancer. Menstrual history is taken which includes regularity, cycle length, flow,dysmenorrhea and LMP. Details of previous and current contraception are noted with regard to compliance ,problems encountered and her expectations. Enquiry is made about episode of unprotected intercourse and intake of emergency contraception. Sexual history is taken which consists of number of partners and change of partners in the last one yr and past history of STI to identify her risk for STI and  Family and personal history is taken about medical illnesses such as diabetes, VTE, MI and breast cancer and endometrial carcinoma. She should asked if she has migraine with aura and acute liver disease.Information is obtained about liver enzyme inducing medication for eg: anti epileptic drugs like phenytoin and carbamazepine. Her future plans for conception are noted. If she is a smoker, details asked about her smoking habit. Examination is undertaken to detect gross varicose veins, hypertension.  Abdomen and pelvic examination is carried out to exclude obvious pelvic masses. B) Morbid obesity is an independent risk factor for MI and VTE. The risk of VTE and MI is further increased with COC use which about 4 fold. COC poses unacceptable health risk for her and can not be recommended. Contraceptives containing progesterone can be used with obesity. They do not increase the risk of cardiovascular disease or VTE but they are associated with mood changes, weight fluctuations and  irregular bleeding pattern. Progesterone only pills are effective contraceptive method with consistent use with a failure rate of 0.3 per 100 woman years. They are not associated with decreased efficacy in obese women and there is no delay in return of fertility after discontinuation.  Most commonly used injectable progesterone only contraception is DMPA given as IM injection every 12 weeks  with a failure rate of less than 4/1000 over 2 years.which is associated with a risk of weight gain, delay of return of fertility after stopping and minimal reduction of bone mineral density after long term use. Progesterone implant containing etonorgestrel is a sub dermal rod licensed for 3 hrs use with low failure rate( less than 1/1000 over 3 years). Cost effective and fertility returns immediately after removal. C)Intrauterine contraceptive methods consist of copper bearing IUD, framed or unframed device and LNG IUS they are effective methods of contraception licensed to use for at least 5 years with a low failure rate of 1-2% for 5 years. Their contraceptive action is mainly local through the effect on the endometrium so not effected by obesity. Associated with return of fertility immediately after use. LNG IUS has additional advantage of controlling idiopathic menorrhagia and protection against endometrial carcinoma. Insertion of intrauterine devices may be technically difficult due to obesity and nulliparity particularly with LNG IUS.
contaception s Posted by sofia  S.
Ans a:I will take detail history of her previous contraceptive use including difficulties with compliance, side effects and reasons for contraceptive failure. Will ask her about her last menstrual period to rule out pregnancy ,regularity of cycles associated gynecological symptoms like dysmmenhorea, menorrhagia,premenstrual tension. Ask her about previous obstetric history and plans for future pregnancy.ask her about her sexual history ,previous treatment of STI and number of sexual partners.drug history to assess possible drug interactions including intake of an enzyme inducing drug .history of hypertension, VTE, liver disorder vascular ds ,diabetes,stoke and cancer of breast endometrium and ovaries. Family history of thrombophilias. Personal history of smoking. Examination will include measurement of BP by an appropriate cuff size. Ans b: hormonal contraception available for her are progesterone only methods as combined preparation containing estrogen are UKMEC 3 where risk outweighs the benefit in view of her high BMI. None of the methods are 100% effective. progeterone only method provide contraception by thickening of cervical mucous thinning of endometirum and may inhibit ovulation. Options available for her are progesterone only pill. More than 99% effective if taken properly.failure rate of .3-.8/100 women year..advantages are that its effective,easily reversible and no delay in return of fertility after stopping pill. Noncontraceptive benefits include relief in symptoms of dysmenorrhea and menorrhagia Disadvantages include side effects like bloating, breast tenderness, headach ,weight gain and irregular bleeding. Also increased risk of ectopic in case of contraceptive failure.need for compliance and pills to be taken at same time of the day(widow period of 3 hrs , 12 hrs for cerazette ). Adv should be given about missed pill. Second option for her is injectable depo medroxyptogesterone acetate.injection to be taken every 12wks so not user dependent.highly effective with failure rate of .25-.5/100 women year. Dose not effected by BMI. Lesser risk of ectopic and PID compared to POP. Disadvantages other than side effects due to progesterone(once injected effect cannot be reversed) include menstrual irrregilarities and ammenhorea with repeated dose. Delay in return of fertility upto 12 mnths. Associated with 2-3 kgs of weight gain.also there is risk of osteoporosis which is only partially reversible after discontinuation.re evaluation of risk and benefit if used more than 2 yrs. Third option for her is implanon (progesterone implant) which is licenced for 3 yr use.advantage ishighly effective with failure rate of0.1/100 women yr.long acting and not user dependent.return of fertility is rapid. Lower risk of ectopic compared to non contraceptive user.no significant effect on BMD. Can be used in women with BMI >30 without reduction in efficacy. Disadvantage is irregular bleeding and procedure for insertion and removal is invasive. Fourth option is progesterone containing IUS.Highly effective,licenced for 5yrs and locally acting. Improves dysmenorrhea, menorrhagia reverses endometrial hyperplasia. Cost effective even if used for 1 yr. rapid return of fertility. Disadvantages include progesteronic effects,irregular bleeding ,ammenhorea,functional ovarian cyst and difficulties with insertion, risk of perforation and expulsion. Written information to support verbal counseling. Ansc:intrauterine contraception for her LNG IUS and copper containing device. She should be screened for Chlamydia and gonorrhea. .contraindication like unexplained vaginal bleeding, active STI, pelvic TB, severe thrombocytopenia,irregular cavity and endometrial cervical cancer awaiting treatment should be ruled out. CuT 380 licenced for 8 yrs failure rate of 1.4- 2.2/100 women yr and LNG IUS for 5 yrs with failure rate 1.1/100 women yr..should be fitted within first 5 days or any time if pregnancy can be ruled out. Barrier method to be used if intercourse within previous 7 days.insertion can be difficult and may reqiure insertion under anesthesia. Disadvantages include perforation 1.2/1000 insertion,risk of STI in first 20 days expulsion specially in first 3 months, pregnancy if occurs more likely to be ectopic. Irregular bleeding. Heavy periods and dymenorrhea with copper iucd. Advise double dutch method if high risk of STI.. follow up after next periods (3-4 wks). Rapid return of fertility after removal. Provide written information and option of alternative methods.
Posted by C M.

Version:1.0 StartHTML:0000000167 EndHTML:0000005710 StartFragment:0000000454 EndFragment:0000005694

A

The most important issue here is to get as much of a history from her regarding her own medical history as well as that of family history. I would need to assess what methods of contraception she has tried and who has been monitoring her contraception – has she been on any hormonal input, or using barrier methods or withdrawal. It is important to asses possible reasons of failure of contraception because it may well be all she requires is adequate education regarding contraception. It is also important to asses her feelings towards contraception and if she has sought advice to date. Has she got regular periods and does she suffer from imtermenstrual bleeding, post-coital bleeding or dysmenorrhea/dyspepsia. A full medical history is important as we know that she has a BMI of 42 she may well have other co-morbidities that would further impede other options of hormonal input. Has she got diabetes or any history of thrombosis or pro-thrombotic blood disorders that she may know about. Does she suffer from migraines. A past family history of thrombosis, migraines, cancers as these are important to assess. A history of smoking is also essentialA clinical examination should be undertaken eliciting any structural abnormalities. A blood pressure is also vital.

 

B

The progesterone only contraceptive pill has been recommended as first line management in morbidly obese women only as we know that risks of developing thrombosis is rare compared to the combined oral contraceptive pill. In this young woman her risk of a VTE is increased 4 fold if she were to have the COC and be morbidly obese. At present we are still being advised that with adequate information and counselling the COC can still be offered to her if that is what she wishes. She would need regular surveillance by her GP who we would recommend she has regular BP and review regarding the COC. If she were ever to forget to take a pill or develops a gastroenteritis that would result in her vomiting the pill back up she should be advised that retaking a pill as soon as possible to when she remember is advised and may need to have barrier method cover for up to a week. Injectables (Implant Dep-provera) are also suitable alternatives that do not increase her risk of VTE though she should be counselled regarding the probable side effect of erratic bleeding that may arise from injectable contraception. The other option available to her is the intra-uterine system which can be placed either in the community or in hospital that would provide contraception for 5 years and could also reduce her periods. She would be counselled not to worry if she has no periods and that the hormones would work locally and a very small amount would get into her system. A copper coil is also another option that is available to her.

 

C

The IUS is not usually the first line of contraception in a nullliparous woman but in this lady it would be a really good option. It could be placed in the community and would last for 5 years. She would need counselling that she may notice that she may become amenorrhoea and that is the effect of the IUS.There are risks associated with placing it namely pain, infection and bleeding likewise risk of perforation of the uterus and injury to any internal organs.  The mirena is effective straight away and the hormonal input into the system is low and so risks of VTE are also reduced to her own background risk. She should be aware that she can always asses that the IUS is in place and that if she has any concerns if infections, discomfort it could also be removed. 

 

H H Posted by H H.

H H

  1. Will take history regarding her previous contraceptive failures resulting in unplanned pregnancy, the type of contraceptive being used, compliance with intake ,perfect or typical use in pill taking, missing  pills and will see regarding her will to take the contraceptive daily or need for long acting reversible contraceptives. Will ask if had any side effects from the contraceptive she was using.

Will take menstrual history,menarche, LMP, period coming every month or is irregular, heavy period and if there is dysmenorrheal. Will ask of sexual history,frequency,number of partners, previous sexually transmited infection which was treated and ask if she used condom or local barriers to protect herself.

Will ask of medical history,previous venous thrombosis,thrombophilia, diabetes mellitus,hypertension,epilepsy,porphyria and any medications as antiepileptic drugs and antibiotics that can lower the efficiency of some hormonal contraceptives. Will ask of personal history of breast cancer. Will ask of family history of thrombophilia ,hypertension and breast cancer. Will ask of social history,partner co operation, smoke history and alcohol intake.

Will ask of measures used to reduce her weight and did she follow diet and if she had bariatric surgery.

Will examine her for BP, see if any respiratory embaressment as she is morbidly obese and see if has varicose veins.

B )As she is morbidly obese COCP are not advised because of risk of thrombosis. Progestogen hormonal contraceptives can be given. As she had previous contraceptive failures I would not prescribe progesterone only pills which has high failure rate in case of missing a pill . I would give her long acting reversible hormonal contraceptive(LARC)( injectables or implants)

Depot provera (medroxy progesterone acetate) can be given every 12 wk ,but there is risk of putting on weight in such obese woman(grade3 obesity), irregular bleeding during 1st3-6 months of use that later disappear with continued use, amenorrhea,bloatedness and headache,but it is an effective contraceptive with  failure rate less than that of tubal sterilization .There is risk of delayed return of fertility when it is discontinued.

Implanon is LARC which is a single rod containing 68mg etogestrel applied on the inner side of upper arm and can last for 3 years. It is a highly effective contraceptive but has the the risk of producing irregular bleeding that can cause concern and patient worry.Efficiency not affected by body weight.

Patches containing estrogen and progesterone(norgestimate derivative) EVRA can lead to risk of venous thrombosis as COCP and not advised in this patient.

          Ther are injectables that contain progesterone derivative that can be effective for one month Norstirate and can be used in this patient

 

C) Intra uterine contraception in the form of non-hormonal copper devices as copper T38o (efficient for 8 years ), multiload copper 375(effective for 5 years) or gyn fix(less expulsion rate),or hormonal devices MIRINA ( contain levo norgestril 52 mg release 20micro gram /daily effective for 5 years ,mainly if patient has heavy periods or dysfunctional uterine bleeding), can be used in this patient. As she is morbidly obese there is increase risk of complications at its application. An experienced person to apply, good positiong of patient on table .The gyn bed should has spicifications for obese lady.Use a large Cusco speculum and good identification of anatomy. There is risk of failure of application and of perforation of uterus. Before application she should be screened for infection,swabs taken and proper antibiotic given.

She is given written information regarding the coil, risks and benfits and properly counseled. Mirena might be difficult to apply and so is not the 1stchoice for intrauterine contraception unless the patient has heavy periods. She is warned that with Mirena she might have irregular bleeding for the 1st6 months of use. Woman choice and wishes are respected

  

 

 

Posted by Bee Fong C.

 

(a)

A detailed history, examination and investigation should be taken before any contraception is given. A detailed history will include her menstrual history. I will ask her the cycle of her period, the heaviness of her period, any dysmenorrhoea, the duration of her menstrual period. Any other vaginal bleeding besides her menstrual period and any other pain besides dysmenorrhoea need to be asked as well. I will ask about what contraceptions she had tried in the past. What sort of problems have she had with the previous contraception? Did she have any side effects from the hormonal or non hormonal contraception? I will need to find out if she is protecting herself from sexually transmitted infection by wearing condoms on top of the contraceptions she was using. Depending on where she lives, I ask about her cervical smear (Wales or Scotland). I will ask about her personal and family history including any previous venous thromboembolism, thrombophilia, SLE, inflammatory bowel disease, headaches with/without aura, smoking history, hypertension, ischaemic heart disease, stroke, any form of cancer (breast, cervical, uterine or gestational trophoblastic neoplasia), epilepsy, liver disease. Is she on any long term medication or was she on any medication (antibiotics, antivirals, antifungal, antiepileptic, St John’s wort) when she fell pregnant?

 

Clinical examination is important. As her BMI has been measured, I will measure her blood pressure and heart rate. Abdominal palpation will review any pelvic masses (may be difficult due to her BMI) ie fibroids, tenderness and speculum and vaginal examination may review abnormal discharge, cervical pathology, and opportunity to take smear (if not uptodate) and triple swabs.

 

I will also order a transvaginal ultrasound to look for any uterine pathology should we decide on intrauterine devices.

 

(b)

There are two types of hormonal contraception – user dependent and long acting reversible contraception (LARC) including the Nuvaring. The user dependent contraception are the combined method (oral, patches) and progesterone only pill (POP). They have a typical usage failure rate of 5%. The combined methods are not suitable for this patient as she has a BMI of 42 (UKMEC 4). She is at a high risk of VTE and cardiovascular disease. The combined methods if not contraindicated will help to regulate and lighten her periods.

She can use the POP if there is no other contraindications ie liver malignancy or breast cancer. Cerazette can be used as it has 12 hours window of taking the last missed pill. POP can have a higher typical failure, which decreases with age and long term usage.

LARC will be more suitable for her if she is not compliant. For example, intramuscular depo provera, intrauterine systerm Mirena and progesterone implant.

 Typical failure rate of depo provera is 0.4. It is an intramuscular injection of medroxyprogesterone given every 12 weeks. It prevents ovulations. Therefore it can cause reduced bone mineral density. There is a delay to fertility for up to 6 months.

The IUS releases 20mcg of levonogestrol daily. It is also licensed as a first line treatment for menorrhagia by NICE. It is licensed for 5 years. However, it is not suitable if the cavity is distorted. It acts by preventing implantation. Some women also have ovulation inhibition. There is a risk of perforation, expulsion and pelvic infection in the first 3 weeks. It has typical usage failure of 0.1%. Progesterone implant is currently the most effective contraception available, with typical failure of 0.05%. It is inserted subdermally 10cm above the medial epicondyle and changed every 3 years. It acts by inhibiting ovulation.

 

Side effects of progesterone methods are mood swings or disturbance, breast tenderness, increase in appetite, and irregular bleeding. One third of women will have lighter periods, one third of women irregular periods and one third of women will be amenorrhoiec in depo provera and implant usage. Irregular periods in Mirena coils can persist for 3-6 months and can be the reason women ask for removal of Mirena and implants. Therefore, it’s important to advise them it may happen.

 

It is important to provide information leaflets on each of the type of contraceptions.

 

(c)

There are non hormonal and hormonal intrauterine devices. They act by inhibiting implantation. They can be inserted in nulliparous. There are risk of expulsion, perforation and pelvic infection in the first 3 weeks.

 

The non hormonal IUD is made of copper on its arm with typical failure of 0.8%. It is licensed for 5 or 10 years depending on the market. The FSRH recommends the use of T380S due to the long duration of action. If it cant be inserted due to narrow cervix, other copper coils can be inserted. Women tend to discontinue them due to heavier menstrual period. It can also be used as emergency contraception.

 

The Mirena IUS is licensed for 5 years. There can be irregular bleeding for 3-6 months.A minority of women also stop ovulation. It reduces the risk of ectopic pregnancy.

 

Both methods can be fitted in the first 5 days of periods. Copper IUD gives immediate protection while Mirena IUS takes 7 days to work, hence extra barrier protection is needed if fitted at other times.

ANSWER TO ESSAY 341 Posted by DHARSHITHA J.

(a) The clinical assessment includes detailed history, complete examination and investigations, if required.

History includes, proper reason for the requirement of contraception. (eg. for contraception alone or as protection against STI) and her future fertility scopes(when she expect the next pregnancy) and need of long term contraception,previously used methods and reasons for previous failures, including side effects.

In addition her past medical history- history of medical disorders such as diabetes, hypertention, thrombophilia, and venous thrombosis, also history of pelvic infection and Sexually transmitted infection(STI) and other  gynaecological factors such as dysmenorrhoea , menorrhagia and  also exclusion of any undiagnosed vaginal bleeding.

Also social history including smoking, family history of breast cancer and thrombophilia  important.

Examination includes blood pressure measurement, exclusion of any breast lumps,and assess for any features of polycystic ovarian syndrome such as hirsuitism, acne. Also pelvic examination to rule out any undiagnosed pelvic masses and to ascertain the size and direction of the uterus.

Investigations -only if required such as screening for diabetes,antiphospholipid antibodies,and thrombophilia screen only if high risk. Any feeling of pelvic masses during examination, require an u/sscan to confirm it.Needs to consider screening for STI if history of multiple sexual partners. And needs to exclude pelvic infection by vaginal swabs prior to intrauterine devices.

history taking should be in a sensitive holistic approach.

(b)There is a wide variety of hormonal contraceptions for her consisting, oral forms, injectables,implants and hormone impregnated LNG-IUS coil.

Combined oral contraceptive pill(COCP) is effective and widely used , but increase the risk of complications such as thromboembolism, breast cancer especially with high BMI. And there is no protection against pelvic infection and STI. Therefore COCP best avoided in this lady considering her BMI. However if she has any signs of PCOS, such as hirsuitism, Dianette would be a good option for her.

Progestogen only pill(POP) a good alternative, and no evidence, regarding reduced efficacy in obese patients, but can cause irregular bleeding and undesirable side effects such as breast tenderness, bloating. Weight gain is controversial with POP. Also failure rates are higher than cocp.

Cerrazette is a new POP that can be used as an alternative which has Desogestrel, but the risk of thromboembolism is higher.

Progestogen implants can be used as effective long term contraception for three  to five years, and no reduction of efficacy in obese patients, and reversible immediately, also gives protection against STI by making cervical mucus thick. But it needs skill to fit it and can cause irregular bleeding.

Injectable progestogens (DMPA) causes significant weight gain in obese patients, also causes delay in return of fertility, osteoporosis,and irregular bleeding and should not recommend in this young lady unless other methods not available.

LNG-IUS another long acting contraceptive method generally for three years, that also effective against menorrhagia.But needs skills for application, also risk of pelvic infection, uterine perforation, expulsion can occur. In addition it can cause irregular bleeding in first few weeks after application and formation of functional ovarian cysts. It should not be inserted in presence of active pelvic inflammotory disease

Progestogen only emergency contraception is not a method to be used in this patient, and should not be offered.

(c) Available intrauterine contraceptive methods(IUCD) include Cu-T, Gynaefix, and LNG-IUS(mirena coil). All IUCDs are effective but having the risks of expulsion, perforation and risk of introducing infection at the time of application,and needs screening for pelvic infection or use of antibiotics at the time of application. They all require training and skills for application.They all are reversible immidiately after the removal, and no impact on the long term fertility.

Cu-T is the commonly used method can be left for nearly  for 8 years, also can be used as an emergency conraception method, but  uterine cavity should be regular, should not be distorted. may be difficult to insert in nulliparous lady without dialation of the cervix  under anaesthesia, relatively it increases the risk of ectopic pregnancy, compared to other methods. It should not be used in the presence of active pelvic infections.

Gynaefix a flexible IUCD, can be used even the uterine cavity is not regular,same efficacy as Cu-T, but less expulsion rate.

Mirena  is coil highly effective ,comparable to the efficacy of male steralization(1:2000), also having non contraceptive benefit of treating menorrhagia, endometrial hyperplasia at the same time and can be left for 3-5 years. However irregular bleeding in first 3 weeks, and possibility of functional ovarian cyst assosciated  with LNG-IUS.

However the selection of suitable IUCD depend on patient ,s willingness to use long term contraception, efficacy,availability of trained skilled clinician,presence and risk of pelvic infection, risk of expulsion , regularity of the uterine cavity, and the presence of other gyn, factors such as menorrhagia, dysmenrrhoea and side effects of that individual IUCD.

 

 

ali naveed haq Posted by Ali Naveed Haq H.

A  detailed  history should be taken regarding her present requirment  and duration for contraception. Presence of comorbidities, e.g diabetes mellitus, hypertension, migraine (cresendo type) cardiac disease, Venous thromboembolism,  smoking, alcohol abuse, infections and  depressive illness may be sought. In the family history history of breast carcinoma, or other malignancies, history of thrombophilia, in providing a clear picture of the womans suitability for contaception choices. Social history use of drugs , smoking number of cigerrattes, alcohol must  be taken. History of three unplanned pregnancies at the age of 24 years and a BMI of 42 Kg/m2 also pint towards an inactive life style ,eating disorder or a pschological /psychiatric illness, maybe a depressive illness,or a bipolar disorder a care ful history may give pointers for a psychiatric evaluation. The lady`s educational background , general intelligence level, social and ethnic status will provide insight into future compliance of usage. The menstrual history relevent to menorrhagia , oligomennorhea, associated PMs, dysparunea, dysmennorhea need to be assessed.

On examination a complete general physical examinationn including vital sign assesment,  thyroid examination, signs of hypothyroidism, breast and abdominal examination , varicose veins. The above may point towards hypertension, hypothyroidism which need treatment and refferal to a  medical specialist. A pelvic examination with screening for sexually transmitted diseases, other pathology needs to be performed. Any suspicious vaginal discharge should be investigated and treated , refferal to a GUM clinic for further management and contact tracing is important in prevention of pelvic inflammatory disease. 

b) Amongst the hormonal contraceptives there are three main choices for this lady a) Low dose combined oral contraceptive pills, B) Implanon, |c) injectable progestogens.

If there are no comorbidities event hen weight alone predisposes this lady to develop complications of venous thromboembolism,  However not being an absolute contraindication to the prescription in m y opinion it should be avoided ,and if for some reason considered than ist and second generation progestogens like norethisterone and levonorgestrel which have a safer risk profile and lower incidence of VTE as compared to dosegestral the fourth generation progestogen comined pills ( YASMIN ) are contraindicated. The benifits of COCs being highly effective , completly reversable easy to administer and highly effective in providing additional benifit of protection against development of ovarian cancer and endometrial cancer are to be considered , in addition protection against endometriosis. She may find these useful in the presence of risks of above condition. The above benefits are over shadowed by the raised BMI which in itself predisposes the woman to develop VTE, hypertension , coronory heart disease and dyslipidemia, diabetes . This may add to her problems .

Intradermal implants can be an effective option but these are devices the have to be placed under the skin under local anaesthesia, their removal can sometimes become difficult , inaddition efficacy of progesterone and dosage in thios lady may be difficult considering the increased body weight, they may cause irreguler bleeding and poor cycle control which may be unacceptable to the woman.

Injectable progesterone like 8 weekly Norigest , or the long acting 3 monthly preperations can be used but these may cause menstrual irregularities , poor cycle control, prolonged ammenorrhea and may take 18 months for complete reversal. The associated weight gain , water retention and depressive symptoms may just be unacceptable to the lady.

 

c.In this lady in the absence of infection, uterine fibroids distorting the uterine cavity intra uterine contraceptive devices may just be the best possible option . However these are associated with increased menstrual blood loss, infection, expulsion of the device, and infertility due to pelvic infection.  Their action is locally mediated and there are no systemic symptoms therefore this lady`s ultimate choice of IUCD may be the best as these are highly effective in preventing a pregnancy with low failure rates.

The choices can include Cu containing devices like Cu T or Multiload, these can be put for five years and donot require a frequent follow up except in cases of sideeffects of mennorhagia, infection, misplacement pelvic pain etc.

Levonorgestral containing Intrauterine device (Mirena ), is a use ful and probably an excellent and highly effective contraceptive option , it can be easily fitted in the uterus and can last for five years, its effects are reversable , and in addition if there is history of mennorhagia this reduces the bleeding .   It hardly has systemic effects therefore is a suitable option of contraception. 

 

 

 

Essay 340 answer Posted by DHARSHITHA J.

Hi Paul,

I have already sent my previous answer(Essay 340) in word format to your e-mail(webmaster@busyspr.com), but still I haven,t received it corrected .

Could you please check it for me.

Thank you.

Dharshitha

 

 

Posted by Rebecca S.
My clinical assessment would begin with a thorough history. This would include: previous contraceptives used to illicit whether she has used contraception before; if she had whether she had problems with concordance and why including relevant lifestyle factors such as her job; and if not what her ideas, concerns and expectations about contraception are. I would explore whether she is currently in a relationship and whether she is planning to start a family in the near future. I would ask about her menstrual history, particularly her cycle and whether she has menorrhagia, dysmenorrhoea or pre-menstrual syndrome. I would enquire about any history of screening for sexually transmitted diseases. I would then enquire about her past medical and surgical history (particularly any cardiovascular problems and any history of venous thromboembolism); family history (particularly breast and gynaecological malignancies and venous thromboembolism); drug history including allergies and social history (particularly smoking status). Next I would examine the patient. This would include weight if not measured recently; blood pressure; abdominal examination and possibly pelvic examination depending on the symptoms elicited in the history. If I performed a pelvic examination I would offer to perform triple swabs to exclude sexually transmitted infections. I would then take consent to perform a pregnancy test to exclude pregnancy. I would then consider further investigations depending on her symptoms and signs which might include a full blood count to assess for anaemia. I would then discuss alternatives with her outlining the risks and benefits, including the non-contraceptive benefits. I would provide written information to support this process. I would explore any ideas or concerns she had about the different methods. I would then support her making a decision and make a suitable plan to provide her chosen contraception, including offering her a barrier method if she had to wait for example for an implant/ intrauterine device and to prevent her from catching a sexually transmitted disease. I would then arrange appropriate follow-up. The options for hormonal contraception for this patient include: the progesterone-only pill; long acting reversible methods of contraception including the contraceptive implant and the mirena intrauterine system; and progesterone-only injectibles . She would not be suitable for the combined oral contraceptive pill (or other oestrogen-containing contraceptives) as her BMI is over 35 which puts her in the UKMEC 3 category with the risks out-weighing the benefits. The progesterone-only pill (POP) is efficacious in women with a raised BMI and the pearl index for failure is 0.3-0.8/ 100 women years offering her good protection from another unplanned pregnancy. She would need to remember to take a pill every day at the same time and, other than for cerazette, would have a three hour window for this (12 hours for cerazette). Some women are concerned about weight-gain but the FSRH guidance 2008 reassures women there is no associated weight gain. The POP is also reversible with no delay in fertility should she wish to start a family. Non-contraceptive benefits include reduced pelvic pain and pre-menstrual syndrome. Disadvantages may include an irregular cycle, breast-tenderness; acne; bloating; reduced libido and vaginal dryness. Progesterone-only contraceptive implants are also efficacious in women with a raised BMI with a pregnancy rate of less than 1/1000 over 3 years overall. They are long-acting (three years) which is useful for women who forget to take a daily pill. They reduce the risk of ectopic pregnancy ompared with non-contraceptive use. Side-effects include an irregular cycle. They are also rapidly reversible and can be inserted/ removed in an out-patient setting. Progesterone-only injectibles would also be another option. They are given every 12 weeks so also reduce the need for compliance. They are effective and reduce the risk of ectopic pregnancy compared with the POP. There is some associated weight gain which would be a disadvantage for this patient. They also reduce bone mineral density so the woman would need to be counseled about an increased risk of osetoporosis. Bleeding can also be heavy and irregular. The mirena IUS will be discussed below. Intrauterine contraception: There are two options for intra-uterine contraception. First the mirena intrauterine system (IUS). This is an effective long-acting reversible contraceptive which lasts for five years. It contains a low dose of progesterone which thins the endometrium and prevents implantation. It also thickens cervical mucous. The non-contraceptive benefits of the IUS include: lighter bleeding (although this may be irregular); reduced pain; reduced risk of ectopic pregnancy and cost-effectiveness. The other alternative is an intrauterine device which is a metal device with no hormone which prevents pregnancy by preventing implantation. This is also effective and long-lasting (5-10 years). However these devices are associated with more heavy painful periods. Both these methods can be uncomfortable to insert and there is a risk of uterine perforation and infection. There is also a risk of unwanted pregnancy if the device is expelled and the woman does not realise.
Answer to Q 341 Posted by Neelima  D.

 

While assessing for this patient’s suitability, I would like to keep into account herpreference and individual concernsregarding contraception.I would also like to find out any contraceptive history and her reasons for discontinuing. I would like ask her if she is planning for a pregnancy in near future and if she has a stable partner. I would take a detailed menstrual history which would include, cycle length, number of days of bleeding, any irregular vaginal bleeding, intermenstrual bleeding, post coital bleeding and associated dysmenorrhoea. Past ,present and family history of medical conditions such as Diabetes mellitus, hypertension, previous thrombo-embolic disease, ischaemic heart disease, thrombophilias,  liver tumours, migraine with headaches and current breast cancer. I would also like to find out if she smokes and her cervical smear history. History of previous sexually transmitted infection should be taken as well. I would take a detailed drug history, prescribed, complimentary and herbal as many medications (such as antibiotics and liver enzyme inducing drugs) interact with the hormonal contraceptives.

B) Combined oral contraceptive pill is UKMEC 4 for BMI>40due to high risk of venous thromboembolism and cardiovascular events so is best avoided. She can be started on Progesterone only pills. The mode of action is changes in the cervical mucus, which impairs sperm passage. desogestrel (cerazette) has more anovulatory effect. The efficacy is not influenced by the weight of the patient. POPs have side effects of prolonged irregular bleeding, headaches, breast pain, changes in libido and weight.

Injectable long acting progesterone include Depo provera (150 mg medroxyprogesterone acetate) every 12 weeks and Noristat (200mg norethisterone enantate) every 8 weeks. Side effects include prolonged irregular bleeding, oligomenorrhoea, and a delay in return of fertility, loss of bone mineral density with prolonged use and weight gain. Progressive weight gain, due to increasing fat stores and anabolic effect, and other cardiovascular risk factors may be aggravated by Depa-Provera. This should only be used if no other contraception is available for this patient.

Implanon is a sub dermal contraceptive devise, which releases etonorgestrel. The double effect of ovulation suppression and cervical mucus changes ate effective in obese women without loss of contraceptive efficacy. It is effective for 3 to 5 years. There is no evidence to change the implant earlier in obese women. It needs special training for insertion, removal can be difficult and it disrupts the menstrual pattern.

Intrauterine system Mirena containing levonorgestrel is a good contraceptive option for her. There is no difference in its efficacy relating to body weight. It can have pain and discomfort during insertion also is associated with risk of expulsion and perforation.

It also has an added benefit of reducing the menstrual flow.

 

C) The intrauterine contraceptives available are Cu-T, Gynae fix, and levonorgestrel IUD.They are good contraceptive option for over weight women and is not user dependent.but insertion needs training.It is contraindicated in women with distorted uterine cavity with fibroids and current pelvic inflammatory disease.It will be effective from 3 to 10 years, depending on the type of contraception used.There is no delay in return of fertility after removal of the intrauterine devise.

In a patient with high BMI, there will be problems with pelvic examination, assessment and insertion difficulty due to body habitus. In the event of cervical shock and collapse, advanced resuscitation procedure might with difficult and not available in the primary care setting. It is also associated with risk of pain, heavy bleeding, expulsion and perforation.

 

 

 

Posted by BHAWANA  P.

 

 

 

Detailed history needs to be taken about her current, gynaecological,personal, past,family and drug history.This will include which contraception was she on every time,any compliance issues or may be low dose of progestrogen causing failure ( though DFSRH recommends that same dose can be used in high BMI patient, cases need to be indivisualised).Her Gynae history will include about regularity of her periods,length of cycle, any associated problems-dysmenorrhoea, menorrhagia, endometriosis, any gynaecological malignancy and history of STIs.

Present/Past history about diabetes and its complications,hypertension,migraine with/without aura, breast cancer,thromboembolic disease, cardiovascular disease, stroke,liver/gallbladder disease, SLE,thrombophilia, antiphospholipid syndrome need to be elicited.

History of smoking in current/past history with number of cigerettes and duration of smoking need to be taken.Also need to know whether she has multiple sexual partners or she is in stable relationship.

Also family history of breast and gynaecological malignancy and if present any carrier of BRCA gene should be asked in history.It is important to ask about any long term medications which can interfere with contraception.

Her examination will include-BMI, BP and pelvis examination to rule out any adenexal mass or large fibroids.(though difficult in raised BMI).

b) OCPs will be unsuitable for this women as she is BMI of 42. This will put her at increased risk of thromboembolic disorder, CVA, malignancy, and should bot be prescribed considering there are no other contraindications.

POPs can be prescribed to her but due to previous history of failure of contraception 3 times, this will not be ideal choice. She need to be counselled that this needs to be taken daily with narrow window period of forgetting the pill. Also she needs to be counselled about side effects of progesterone which include acne, headache, weight gain( can be a concern in her case),irregular bleeding.

Implanon is Progesterone only method which lasts for 3 years and is long acting reversible contraception. Though efficacy is quite good, return of fertility might take time ( upto 1 year).She should be counselled about side effects of progesterone. 

Injectable progesterones needs to be given  regularly either every 8 weeks/ 12 weeks with Intamuscular injections. There are concers about osteopenia in young women though efficacy is quite good.She should be counselled about side-effects of progesterone.

Mirena coil is progesterone releasing hormone coil for which the contraceptive effect lasts for 5 years. Its failure rate is very less .It is one of the most efficaceous contraceptive. The reversal fertility is immediate once coil is removed.She needs to be counselled about progesterone side- effects.

Copper coil is the most efficaceous contraceptive lasts for 5-8 yrs depending on the type of cu-coil used.It does not have any side effect of progesterone.It has side effects of causing dysmenorrhoea and heavy periods.

Barrier contraception should be advised as well to her if she does not want any of these contrception and if she has multiple sexual partners .

c) Copper intrauterine device acts by inhibiting fertilization and is toxic to sperm. It's failure rate is 0.1%.It is licensed for 5-8 years depeding ont he type.It is effective immediately and return to fertility is also immediate.It does not have hormone content so free of side effects of progesterone as in mirena.The main side-effect is dysmenorrhoea and heavy peiods.

Mirena intrauterine device acts by preventing implantation and to some extent ovulation.Efficacy is 0.2-0.6%.It is licensed for 5 years.It can cause irregular or heavy bleeding for 4-6 months and patient need to be counselled for that.It has side effects of progesterone.

The risk of ectopic pregnancy is less than women not using any contraception for both of them.Both of these can be used in this women successfully with raised BMI and would be good option in this women.

VANEEZA K ESSAY 341 Posted by vaneeza K.

 

A)Patient will be assessed clinically by enquiring detailed history followed by clinical examination.Previous methods of contraception used,percieved reasons for failure,reasons for discontinuation and her failure of reproductive intentions should be discussed.Detailed menstrual history including age of menarche,cycle length,flow,regularity and associated dysmenorrhoea will be disscussed.sexual history including previous STIs and treatment,number of sexual partener and change of sexual partenes during last 6 months should be asked.Personal and family history of VTE,cardiovascular diseases,breast,ovarian , endometrial cancers and osteoporosis are relavant and to be disscussed with the patient.Any drug history that can effect efficacy of contraception like anti tuberclosis should be evaluated.History of smoking ,number of ciggerates smoked per day should be asked.Her blood pressure would be checked and legs would be examined for any gross varicose vein

B)As her BMI is high.And COCPS are associated with 2-3 fold increased risk of VTE , it is not suitable mathod for her.

Progesterone only pills are highly effective with failure rate of 0.3-0.8/100 women year,reversible with no delay in return of fertility.Its efficacy does not decreased with high BMI but has to be taken daily at sametime so compliance will be problem especially in patient with repeated failure.Also associated with progesterone side effects bloating,amenorrheoa,breast tenderness,acne,decreased libido and vaginal dryness.There is no association between POP use and weight gain.

Injectable progesterone including depo provera given every 12 weeks Noristerat given every 8 weeks highly effective with PI 0.25—5 and 0.4—2/100 women year(Noristerat) with better compliance.It is associated with lower risk of ectopic pregnancy,PID and endometrial cancer.But there is delay in return of fertility 1—2 year after  last injection.It is associated with weight gain of upto 3kg at 2 years and those with BMI>30 are more likely to gain weight.As it causes reduction in bone mineral density so not suitable for                   young patients and patients with risk of osteoporosis. 

Progesterone only implants,implanon,LARC licensed for 3 years use with pregnancy rate of < 1 in 1000 over  3 years.It can be used in women with BMI>30 without reduction in efficacy but associated irregular bleeding,a reason for discontinuation.

LUNG—IUS another option,licensed for 5 years highly effective but reversible.It reduces menstrual blood loss in patients with menorrhageoa,improve dysmenorrheoa with PMS  but associate with expulsion,perforation and functional ovarian cysts.Written information will be provided.  

C)  cu  iucd,immediately effective long acting contraceptive with failure rate of 0.1%.It is reversible with rapid return of fertility.But associated with irregular heavy bleeding,perforation,expulsion with STI  especially first 3 months of insertion.

  1. LNG IUS levonorgestril(20micro gram/day) releasing device licensed for 5years.Along with contraception,it is also used for treatment of menorrhagia,endo metriosis,fibroid(if uterine cavity not distorted)dysmenorrheoa with PMS.But associated with expulsion,perforation,functional ovarian cyst and progesterone side effects. 
neha Posted by anuradha N.
hut
Dr Anuradha R N Posted by anuradha N.
A)clinical assesment includes history- of last menstural period,flow pattern,interval between previous pregnancy termination,meathods used for MTP,previous contraceptive usage,compliance,if non compliant reasons for it should be enquired,inquirey into her medical illness,hypertension,thrombotic(past,recent)episodes,migraine,stroke,myocardial infarction,smoking,liver enzyme inducing drugs,family history of thrombophilia to be done.sexual history of change in the partner in last 1 year or partner having other partnern to be made out as she could be at high risk of sexual transmitted diseases. EXAMINATION her BMI,blood pressure to be taken . detailed examination of breast ,abdomen, pelvis not indicated unless her clinical condition needs such an evaluation.i would like to assess her psycological aspect of choice of contraception, her preferences , meathods available , failure rate with each meathosd , reversibilty , risk assosiated with each meathod should be explained B)the harmonal contraception available are combined oral contraceptives (COCs), patches ,vaginal rings , implants,depot injections,LNG ius ,emergency contraceptive pills(LNG pills).COCs NUVA ring ,EVRA patch which contain both ethynyl estradiol and a progestogen pose unacceptble health risk ( UKMEC4) for increase risk of thromboemvolism nd MI for her BMI 42kg/m2 . POPs can be started immediately after abortion less than 48 hrs or day1 to 5 of cycle with one pill per day over 3 wks and starting the next pack in next cycle. it acts primarily by thicking of cervical mucus and inhibiting ovulation . it can be used safely in patients who are obese with good efficacy more than 99 %. pearl index 0.3 to 8 per 100 women years. efficacy is good if used correctly and consistently. cerazzette has the highest efficacy in this group. efficacy is less if the pill is taken more than 3 hrs late than the schedule( cerazzette lag period 12 hrs). other problems with pops include amenorrhea (20%) to bleeding irregularities (40%), mood changes.POPs are not the choice if the compliance is poor .return of ovulation is within 3 months ,no delay in fertility potential progesterone injections (depot provera) used once in 3 months as intramuscular injections act by their ovulation they are highly effective low failure rate less than 4 per thousand. they are the prefered choice if compliance is poor . disadvantages include delay in the fertility ,persistent ammenorhoea, infrequent bleeding,need of injections , bone loss if used more than 2 years . however not associated with increased risk of stroke ,VTE, MI,obesity . subdermal implants (implanon) can be used for 3 years, efficacy <1/1000 women years ,over 3 years. bleeding pattern is comparable with other progesterone contraceptives. main advantage is insertion once in 3 years ,improves dysmenorrhoea , less risk of ectopic pregnancy.it becomes highly cost effective when used ovver 3 years period . LNG -IUS(mirena) has good efficacy contraception effective for , upto 7 years, especially use ful if she has endometriosis ,dysmenorhoea, heavy bleeding. it cannot be used as emergency contraceptive method,not cost effective if she wants return of fertility over months. emergency contraceptive pill LNG 1.5 mg pill are used if she has infrequent cohabitous, failed other contraceptive methods . she should be councelled to use barrier methods if she has high risk for STIs (non monogamous relationships) C) IUC- LARC- CuT 380s ,LNG-IUC are the preferred methods for her . they primarily act by inhibiting fertilization, low risk of ectopic pregnancy. they can be inserted immediately after abortion wit in 48 hrs , do not interfere with her future fertility pattern. mainly used if obese , high risk for stroke, VTE, MI,migraine,poor compliance. no delay in the fertility after removal. highly cost effective, can be used upto 10 years , with less follow up ,doesnt cause amenorrhoea ,irregular bleeding pattern however it can cause prolaonged bleeding for initial 3 to 6 months . risk of PID increase in intial 20 days but not after that .she can be taught to feel the thread , to report if thread not felt, amenorrhic . she should be told about the insertion procedure, risk of perforation 0 to 2.3 per 1000 insetion, risk of expulsion 1 in 20 . as she is obese high risk of cardio-vascular risk,with repeated termination,copper-T s 380s are the ideal and cost effective methods.
essay 341 by NA Posted by nahid N.
I'll take a detailed history to assess her risks associated with hormonal contraception which include history of diabetes, hypertension ,personal or family history of DVT.History of sexually transmitted diseases.Result of her last pap smear.Menstrual history including her last menstrual period and blood flow.Time of her last termnation.The problems faced with past methods of contraception .Nature of job like working in shift duties.I'll check her blood pressue.Look for varicose veins.I'll do her speculum examination and take high vaginal swab for microscopy and culture sensitivity and cervical swab for clamydia. (b)Her hormonal options include (1)progestogen only injectables ,these are depo-medroxyprogesterone acetate 150 mg every 12 wks (pearl index .2-.4 /100 women years)and norethisterone 200mg every 8 wks(peral index .4 -2 /100 women years).It is very effective method of contraception with lower risk of ectopic,lower risk of PID ,and lower risk of endometrial cancer.May cause menstrual irregularity.Delayed return to fetility and weight gain.(2) Progestogen only implants.Very effective method of contraception with pregnancy rate less than 1 in 1000 over 3 years.It is reversible ,reduce overall risk of ectopic pregnancy,with rapid return to fertility.Assossiated with risk of irregular bleeding which is most common cause of discontinuation.(3) progestogen only pills,these are cerazette,noriday,micronor ,norgeston and femulin.Very effective method of contraception with failure rate from .3 to .8 per 100 women years.No evidence that one method is superior to other .Reversible.Assocated with progestogenic side effects,such as bloating,breast tenderness,acne and mood swings.increased risk of ectopic.(4)combined oral contraception is contraindicated for her due to increased BMI. (c)IUCD is an effective method of contraception.Need evaluation before insertion .vaginal and cevical swabs should be taken for microscopy and culture sensitivity and chlamydia.Pelvic examination for uterine size and postion is needed.contraindicated if at high risk of sexually transmitted disease,menorrhagia,or benign trophoblastic disease.Upto 4 wks post partum .First choice IUCD is copper T 380 with low failure rate of .4 to 2.2 /100 women years.low expulsion rate 8/100 women over 5 years.licensed for 8 years.other choices for non hormonal contraception are gynefix and multiload cu 375.Hormonal IUCD is mirena .licensed for 5 years .side effects are irregular bleeding amenorrhoea,and progestogenic side effects -edema,headache,breast tenderness and acne.Overall complications of IUD include small risk of perforation 1.2/1000 insertions.risk of expulsion mostly in firt year.6 fold increased risk of STD in first 3 wks.Increased menstrual loss and dysmenorrhoea.
Hbadran Posted by HAnaa B.

My clinical assessment would include detailed history of accompanied medical conditions to her morbid obesity, like hypertension, diabetes, PCO, gall bladder disease or previous DVT.

History of stable relation for more than one year with her own partner, history of previous children.

LMP, cycle regularity, length and accompanied menstrual symptoms with or without PMS.

History of unprotected intercourse in this cycle and its numbers.

Drugs taken for any disease that may be affecting the use of the chosen method antiepileptic, orlistat warfarin.

Previous Pap smear result

Family history of comorbidities that my affect her conditions, especially breast cancer or endometrial cancer.

Patient reliability and its understanding level.

Investigation, may include

Fbc, glucose tolerance test ,pap smear, pelvic ultrasound

B

Hormonal method of contraception may not be the best choice for morbidly obese lady, but can be offered after explanation of the risk accompanied with its use; especially risk of VTE which is increased by 5 folds the normal population.

Combined OCPs can be offered with caution special 3rd generation ones that is having more risk of VTE in relation to the 2nd and the 4th one.

It should be started from the 2nd day of the menses with regular 21 day use  and 7 days interval monthly, side effect include nausea and GIt upset with the start of use  improved by time, spotting as break through may occur , symptoms necessitate stopping of the method include DVT and migraine headache

Preferably POP can be offered as continuous 30day use. Risk of VTE is not increased in relation to her situation reliability 98 % as for the OCPS, spotting with the start of use day one of the cycle may improve by time; some people would experience amenorrhea with the long use specially if used during lactation.

LARC, method of contraception can be used in case she is not reliable for the daily use of any tablets, like Medroxy progesterone acetate injectable can be used as 3 monthly injections starting day one the cycle .irregular bleeding may accompany its use can be corrected by reassurance or tanexamic in some cases others my not respond to any hormonal treatment warrant the use of the method. Long use may lead to loss of BMD by 6% per year.

Sub dermal implants of progesterone licensed for use for 3 years need expert for insertion and removal. Irregular unpredicted bleeding also is one of its side effects.

Combined hormonal patches can be used one patch weekly for 3 weeks with one week off, not licensed for use in case of weight more than 90kilo as per the company.

Hormonal ring (Nuvaring) can be inserted in the vagina for 3 weeks starting with the menses with one week free. need good motivation and understanding lowest hormonal level only 15micro E.

Levonorgesterl releasing IUCD Mirena is also a hormonal and local method that can be used. In case of heavy menstrual bleeding licensed for use 5 years, not easily fitting nulliparous.. But has other contraceptives advantages.

 

C

Intrauterine device can be used at any time of the cycle, copper contains IUCD can be reliable method used for 5-10 years , failure rate 2/1000 cases, may increase the menstrual flow rate  and the dysmenorrhea. Can be expelled, increase ectopic risk and PID .little risk of perforation with the insertion. 

Mirena system can be offered for use licensed for use for 5 years. Decreasing the menstrual flow and having got extra contraceptive benefits in treating menorrhea, dysmenorrhea, and endometrial hyperplasia. Not easily expelled.

Having got bleeding pattern with the first use lasting from 3-4 month, need good counselling. Amenorrhea may occur after one year of use, may be accompanied by progesterone side effect,

bloating and breast tenderness.  

essay 341 Posted by Yingjian C.

 

a) Firstly I would take a history. I would ask her if she has any history of medical conditions such as diabetes, hypertension, venous thromboembolism, pelvic infections.   I would also enquire about her sexual history, whether she is in a steady relationship or has multiple sexual partners, and her plans to conceive. This would affect my decision to recommend her a hormonal or non hormonal, short or long term contraception and help me determine if she is at risk of sexually transmitted infections. Her menstrual history would be important as well. I would look out for any irregularities in her menstrual cycle as well as any complaints of menorrhagia or dysmenorrhea. If there is a presence of oligomenorrhea followed by heavy menstrual bleeding, I would offer her an option which will protect her endometrial lining. If she complains of menorrhagia or dysmenorrhea, I would avoid options such as intrauterine device. I will also ask if she has a family history of any venous thromboembolism or known thrombophilia, in which case a non hormonal method would be favourable, and if she is taking any medications currently that can potentially interact with contraceptive drugs, such as antibiotics and drugs which can induce liver enzymes so that additional methods such as barrier methods can be utilised if she chooses to use hormonal contraception. I would also find out if she had previously tried any forms of contraception and the reasons for discontinuation/ non compliance if so.

 

Next I would conduct a physical examination, focusing on obtaining her blood pressure and performing a pelvic examination to assess the uterine size and adnexa. This examination is likely to be hindered by her body mass index. If there is any ambiguity, I would offer her a transvaginal ultrasound scan to assess her pelvic organs, especially if I am considering inserting an intrauterine device. I would offer her a Chlamydia swab as a screening as she is in the age group <25 year old where Chlamydia infection is significantly prevalent, so that she can be treated prior to inserting of an intrauterine device.

 

b) The first choice of hormonal contraception I would offer to her is the progesterone implant as this is an effective form of contraception and does not require her compliance, in view of the fact that she had three previous termination of pregnancies. This implant will not affect her weight and is not contraindicated for use given her body mass index. The second choice of hormonal contraception that I would offer her is injectable progestrogens such as depot medroxyprogesterone acetate. Similar to the implant, there is no contraindication given her body mass index. However it has the propensity to cause weight gain, on average 3kgs over 2 years and hence is not so ideal for her. The progesterone only pill can also be offered but I would caution the use of it as it requires good user compliance to the timing of the medication for reliable contraception. The combined oral contraceptive pill should not be offered to her in view of her morbid obesity as it can significantly increase her cardiovascular risks as well as her risk of venous thromboembolism.

 

c) The use of an intrauterine device in her case would be suitable as it is a reliable form of long acting contraception that does not require her compliance. It is also suitable for use in women who are morbidly obese (UKMEC Category 1). If she has any concurrent menstrual disturbances such as menorrhagia, I would offer her the levonorgestrol intrauterine system. However I would caution her regarding need for barrier methods if she is at risk of sexually transmitted infections and pelvic inflammatory disease, for example if she has multiple sexual partners.  

srabani Posted by SRABANI M.

Srabani

  1. A detailed history is very important for this lady. Her menstrual history will include LMP , cycle, length of cycle, regularity, amount of loss and menarche. Her obstetric history will include termination of pregnancies and any complication , fertility wishes. Contraception history will include what methods she was using in the past and she is using at present, what were the problems , compliance. A medical history is important for this lady which includes history of migraine with or without aura, various medical conditions like diabetes, cardiac problem, epilepsy.drug history is very important if she is taking any medication now and also history of allergy should be taken. Family history of breast Ca, osteoporosis, VTE are very important while discussing about contraception.Social history should include smoking , alcohol and drug habit and her occupation.

General examination should include Pulse, BP and BMI. Initial investigation will include Pregnancy test.Trans vaginal scan may be done prior to IUD/IUS insertion but it is not routinely done.

  1. For this lady combined oral contraceptive pill is not suitable and also combined patch ( Evra) or nuvaring can not be used as her BMI is 42kg/m2.For this lady Progesteron only pill ( POP) can be offered. POP  works by thickening of cervical mucous and also by reducing sperm penetrability.For some POP ( like cerazette) it works by suppressing ovulation.She should be advised to take POP around the same time of day .there is no evidence that one POP is superior to other and also two POP are superior to one POP.Advantages of POP are reversible, no delay in fertility,can be used where oestrogen is contraindicated or in breast feeding mum.compliance may be a problem. Side effects are bloating, amenorrhoea, breast tenderness, acne, weight gain, decreased libido.Before starting POP contraindications should be excluded like current breast Ca, current stroke or ischaemic heart disease .

Contraceptive injections can be offered although her high BMI may not suggest injection would be preferred contraception.Contraindications are  multiple risk factors for  cardiovascular disease, current Ca breast, unexplained vaginal bleeding.Injections can be two types – Medroxyprogesteron acetate( given 12 weekly)  and Norethisteron( 8 weekly) .Advantages are effective contraception, lower risk of endometrial CA and good compliance.Disadvantages are weight gain, delay in return of fertility. Menstrual irregularity, heavier bleeding and osteoporosis.

Implanon is another option for this lady.Implanon is a progesterone only subdermal implant containing etonogestrel.Highly effective contraception and it lasts for 3 yrs.Primary effect by inhibiting  ovulation and also prevents sperm penetration. Advantages are very effective , rapid return to fertility,can be used in women with high BMI like this lady.Has got no effect on bone or weight gain.Disadvantage is irregular bleeding pattern.contraindications are Current Ca breast, unexplained vaginal bleeding, pregnancy, current stroke or ischaemic heart disease.

  1. IUS contains 52 mg Levonogestrel, releasing 20microgram per day. Advantages are very effective contraception for 5 years.It can be used in menorragia,dysmenorrhoea, as part of HRT, PID protection and also treatment for endometrial hyperplasia.Disadvantages are irregular period, difficulty in insertion and removal , amennorrhoea, progestogenic side effects, expulsion and perforation .Contraindications are pregnancy, unexplained vaginal bleeding,, active PID,current Ca breast, uterine anomaly etc.It acts on endometrium  leading to endometrial atrophy and prevents implantation.

Copper containing IUD provides long term reversible contraception for 5/10 years.It can be Cu 380 T, T- safe, multiload or Gynaefix. Advantages are low failure rate, no hormonal side effects and  also not user dependant.Mode of action is by foreign body reactionin the endometrium preventing implantation and copper inhibits spermatozoa motility.disadvantages are heavy period, insertion/removal may be difficult, perforation, expulsion., risk of infection, dysmenorrhoea. Contraindicationa are pregnancy, unexplained vaginal bleeding, PID, uterine anomaly and copper allergy.IUD and IUS can be inserted any time of cycle if pregnancy is excluded.It can be fitted upto 7 days from first day of LMP.

 

 

Essay 341 from Helena Posted by Helena F.

A 24 year old nulliparous woman has been referred to the gynaecology clinic because of problems with contraception. She has 3 unplanned pregnancies resulting in early medical termination. Her BMI is 42 kg/m2. (a) Discuss your clinical assessment [6 marks]. (b) Discuss the options for hormonal contraception [10 marks]. (c) Discuss the use of intra-uterine contraception in this woman [4 marks]. 

a)

Plan: History:

Examination

Investigations

Initial assessment of this woman begins with focused history. What contraception has she used in the past and which methods were responsible for her three unwanted pregnancies? Was she using the contraception correctly? Secondly what is her relationship status. If she has had three unwanted pregnancies, she may also have been at risk of sexually transmitted infections (chlamydia, Gonorrhoea, HIV). Has she attended an STI clinic regularly and or recently, and has she been treated for any STI's in the past.  With regards to her menstrual history I would want to know her cycle length and whether she had any symptoms that would influence choice of contraception. The date of her last menstrual period (LMP) and whether she had had any unprotected intercourse in the last month will effect choice of contraception and whether emergency contraception might be required in the case of UPSI within the last 72 hours. In terms of her past medical history and medication history I would want to know about migraine, epilepsy and epilepsy medication, mental health including use of St John's Wort, previous thromboembolism and immediate family history of thromboembolism, as well as diabetes. Examination should include blood pressure, abdomen and pelvic exam including HVS and endocervical swabs for chlamydia and gonorrhoea. I would want to do a urinary pregnancy test, an Hb A1c.

b) Her options for hormonal contraception are restricted to progesterone only methods as her BMI of 42 contraindicates use of estrogen according to UKMEC (United Kingdom Medical Eligibility Criteria for Contraception), due to her significantly increased risk of thromboembolism among other things. Her weight also complicates the use of progesterone only contraception, as it may affect the absorption and efficacy of it. All Progesterone only methods work primarily by changing cervical mucus so preventing fertilisation, but there is one that inhibits ovulation. They are all reversible. There are questions about most appropriate dose in obese patients. They all have a risk of breakthrough/intermenstrual spotting

The Progeseterone only pill may cause inter-menstrual spotting, and its efficacy in obese patients is debatable. many clinicians prescribe 2 pills daily instead of 1 but this is outside prescribing guidelines and there is insufficent evidence to recommend it 

Injectable progesterone eg depo-provera may take longer to wear off. It may cause amenorrhoea

Progesterone implant

Levenorgesterel intra-uterine system (LNG-IUS) 

 

 

 

 

Helena's finished essay for marking please rather than the first version Posted by Helena F.

a) Plan: History: LMP, UPSI, Sexual hx, PMH and medications, Other risk factors, Allergies, Lifestyle

Examination

Initial assessment of this woman begins with focused history. What contraception has she used in the past and which methods were responsible for her three unwanted pregnancies? Was she using the contraception correctly? Secondly what is her relationship status. If she has had three unwanted pregnancies, she may also have been at risk of sexually transmitted infections (chlamydia, Gonorrhoea, HIV). Has she attended an STI clinic regularly and or recently, and has she been treated for any STI's in the past.  With regards to her menstrual history I would want to know her cycle length and whether she had any symptoms that would influence choice of contraception. The date of her last menstrual period (LMP) and whether she had had any unprotected intercourse in the last month will effect choice of contraception and whether emergency contraception might be required in the case of UPSI within the last 72 hours. Is her lifestyle amenable to regular pill-taking? In terms of her past medical history and medication history I would want to know about migraine, epilepsy and epilepsy medication, mental health including use of St John's Wort, previous thromboembolism and immediate family history of thromboembolism, as well as diabetes. Examination should include blood pressure, abdomen and pelvic exam including HVS and endocervical swabs for chlamydia and gonorrhoea. I would want to do a urinary pregnancy test.

b) Plan Progesterone only, POP, Injectables, Implants, Intra-uterine system; Method of action, failure rate, method of correct use, side effects,

Her options for hormonal contraception are restricted to progesterone only methods as her BMI of 42 contraindicates use of estrogen according to UKMEC (United Kingdom Medical Eligibility Criteria for Contraception), due to her significantly increased risk of thromboembolism among other things. Her weight also complicates the use of progesterone only contraception, as it may affect the absorption and efficacy of it. They are all reversible. There are questions about most appropriate dose in obese patients. They all have a risk of breakthrough/intermenstrual spotting, and possibly weight gain which may be highly significant for this patient. Some antibiotics may affect efficacy so she should inform her GP if she has to take any.

The Progeseterone only pill works by thickening cervical mucus preventing fertilisation of eggs. Cerazette may sometimes prevent ovulation (the release of an egg) Its efficacy in obese patients may be affected; many clinicians prescribe 2 pills daily instead of 1 but this is outside prescribing guidelines and this is not evidence-based. The pill has to be taken at exactly the same time every day, within a 3 hour time window except for Cerazette which has a 12 hour window. If one is missed or late it should still be taken but condoms should be used for 7 days from the missed pill. Failure rate 3 per 100 women years Some antibiotics may affect efficacy so she should inform her GP if she has to take any.

Injectable progesterone eg depo-provera DPMA is a 12 weekly injection. It inhibits ovulation. In obesity the interval may need to be less than 12 weeks, but again   It may cause amenorrhoea, there is a risk of delay of up to a year before menstruation, and fertility returns to normal. Failure <1%

Progesterone implants may be inserted just under the skin (sub-dermally) usually under the arm. It can remain in situ up to 5 years and is simple to remove. It may be inserted with local anaesthetic so is minimally uncomfortable. IMplanon or now Nexplanon which has a safety mechanism to ensure the implant is not left in the applicator, and is radio-opaque. Failure <1%

c) Levenorgesterel intra-uterine system (LNG-IUS) is a hormonal intrauterine device approx 3 cm long which sits in the uterus. It has the added benefit of treating menorrhagia if present. It should be possible to insert in an outpatient setting, but obesity may make the technicalities of this somewhat challenging. It can cause cramping on insertion. For up to 6 months it may cause unpredictable/constant bleeding, but this should then settle down.

Copper coils act principally by preventing fertilisation as copper is toxic to ova and sperm. It also prevents implantation. It can also be used as emergency contraception if the only UPSI in the last month was less than 72 hours ago and pregnancy test is negative. 

SK Posted by HAnaa B.
This young lady is morbidly obese in category 3obesity with failed contraception resulted in 3 times Medical TOP.History should be taken for reasons of failure of contraception.Type ,duration & compiance of contraception used.Her plans of future pregnancy.Any history of STD ,recent or recurrent PID,VTE,comorbodities like HTN,DM,any liver disease,Vascular disease,thromboplilia,SLE etc.Smoking habits.Having stable relationship with her partner,any sign of depression etc.Menstrual history should be taken esp.any abnormal pattern,dysmenorrhea ,Premenstrual tension etc.Family history of VTE,breast cancer etc.Drug history should be taken esp.Antiretrovirals,Anti convulsants & Antimicrobials as they interect with hormonal contraception.On Examination we will check her BP.On Speculum examination check for cervicitis,Swabs taken for STD,Pap smear taken if not done within 2-3 years.Breast should be examined for any abnormal lumps. Combined hormonal contraception including OCP;s ,patch and vaginal ring all are contraindicated & in UKMEC 3 for this morbidly obese lady.Progestogen only methods are good option for this lady.There are 4 kinds.POP's like micronor(norethisterone) and cerazette(desogestrel).They are effective.Failure rate is 0.5/100women/year.Can be used as a tratment of PMS,Primary dysmenorrhea,Endometriosis.Can prevent from PID Should be taken on time (3hrs safety window for all others & 12 hrs for cerazette).It is reversible & no delay in fertility after discontinuation.Has progestogenic side effects like bloating,headcahe,Acne,decreased libido,functional ovarian cysts which is less with cerazette as it inhibit ovulation by 97%.Ameorrhea,mood swings needs to be discussed.Slightly increased incidence of Ectopic.It can be started immediately after miscarriage.one pill needs to be taken everyday so compiance can be a problem for this lady.Other 3 choices are included in LARC.First of all Injectableslike Depo-Provera(MPA) every 12 wks.Itis effective failure rate is 0.2/100womenperyear.Low risk of Ectopic & PID.70%women will have irregular bleeding pattern after one year of use.Fertility is delayed for 1-2 years & Wt.gain which will be unacceptable for this lady.There is decrease in BMD & she is already at risk of osteoporosis.These side effects make this choice less favourable for her. Implants(etonorgestral,active metabolite of desogestral)are effective.Pregnancy rate is less than 1 in 1,000 over 3 years..Decreased incidence of Ectopic,PID.Rapid return of fertility.Disadvantages are amenorrhea in 20% and irregular bleeding pattern in 50%,so needs to be counselled well.Skill is needed to apply it & removal is little uncomfortable.Third choice of LARC is IUS-LNG. c, Intrauterine devices that can be used in this lady is Copper-T,Gynaefix(Needs skill to apply)And Mirena.They are LARC with immediate return of fertility.Mirena for 5 years & Copper t for 8 years so compliance is better.Small Risk of expulsion,failure rate 0.1%with mirena.Pelvic infection in 1st 6 WKS.should be informed.Take swabs & give antibiotics if any sign of cervicitis.Very small risk of perforation.Mirena application can be difficult technically & may requires sedation & cervical dilatation.With copper T increased incidence of heavy menses & with Mirena,incidence of Amenorrhea is 35% after one year of use.Irregular vagional bleeding in 1st 3-6 months with Mirena.patients needs to be counselled about them & written informatin provided & her wishes should be respested
Posted by Dr Dyslexia V.

X

a)      History in regards to past contraception usage would be important . The type of contraception or compliance and contraception failure in spite proper  adherence to it. Her menstrual history is important as the last menstrual period and presence of any recent coitus could lead to a ongoing pregnancy currently. Her cycle length, presence of dysmenorrheal or menorrhagia history could dictate usage of hormonal contraception. Sexual history in regards of the number of sexual partner and usage of condom could infer the risk of sexually transmitted disease(STD). History of any previous pelvic inflammatory disease or treatment is important as well and relevant for intrauterine contraceptive device(IUCD) usage. Underlying medical history diabetes, hypertension with any end organ damage, migraine or venous thrombo embolism(VTE) is important as it is a contraindication for combine oral contraceptive(COC) pill. History of any current drugs usage such as anti epileptic drugs such as phenytoin or antibiotic such as ampicillin which are enzyme inducers which could cause hormonal contraception failure. Social history such as smoking increases the risk of VTE in usage of COC. History of previous cervical disease or previous cervical smear results is important for usage of IUCD. Examination of her should include blood pressure for presence of hypertension. Abdominal examination for presence of any pelvic mass is also as presence of a fibroid which could cause distorted uterine cavity which is a contraindication for IUDC insertion. A pelvic examination to assess the cervix of any suspicious lesion should also be done to assess the difficulty for insertion of IUCD.

b)      Hormonal contraception include COC pills, progestogens such as POP progesterone only pills, injectables such as depot medrooxyprogesterone acetate(DMPA), etonegesterol implant such as implanol. Her known risk factors is currently being morbidly obese with a BMI of 40 makes her UKMEC category 3 for COCP as there is increase in VTE, increase in myocardial infarction and stroke with its usage. POP could be used in her as there is no contraindication to its usage in obesity. It is easily used and it is used continuously without any breaks and easily available with a failure rate of 0.3 per 100 women if properly used. The drawback includes its small window of daily usage time of 3 hours compared to cerazette which has a twelve hours window period makes it difficult to adhere. It has the disadvantage of causing irregular bleed in 40% of the women and amenorrhea in about 20% of women after 1 year of usage. It has other disadvantage which include mood irritability, breast tenderness, bloatedness and depression. This also increased risk of ectopic pregnancy if pregnancy occurs during usage of POP. DMPA are other method which is reliable as it requires less level of commitment as it is a 3 monthly injection. It is however associated with weight gain as these patients are already obese. It is also associated a decrease in bone mineral density and a delay in return of fertility up to a year when discontinued. It is associated with amenorrhea in about 70% of patient after 1 year of usage. The progesterone only implant such as etonegesterol is beneficial as its usage which lasts about 3 years for contraception. It has a rapid return to fertility upon discontinuation. It also reduces an overall risk of ectopic pregnancy compared to the POPs. It has no association with increase in weight or loss of bone mineral density however it is associated with irregular bleeding in which 20% will become amenorrheic and 50% will have irregular bleeding.

c)       There is no contraindication to reuse of IUCD either copper bearing IUCD or LNG intrauterine system in this patient. This device will be ideal for a patient who wishes not to take regular medication and is keen for long acting reversible contraception. The intrauterine device CU380 is licensed  for 8 years used while LNG IUS is licensed for 5 years for contraception. Both has the advantage of rapid return of fertility after discontinuation of use as this patient is young. The risk of perforation is up to in 1000 insertion and it would be higher in this patient as she is obese and it would be technically difficult. The risk of PID is increased in the first 20 days of insertion especially in a lady who has 3 unwanted pregnancies with medical termination. The LNG IUS will be useful to control any idiopathic menorrhagia which could be present in obese patient.

you miss my answer paul Posted by HAnaa B.

Dear Paul ,

Kindlly correct my asnswer posted last week from hbadran posted by Hanaa. i cannot see my name there.

Posted by HAnaa B.

My answer was posted on the 10th of january , you may be missing my answer,

thanks

Essay 341 Posted by Victoria A.

Dear Paul

 

Just wanted to say a big thanks for marking all these essays, I am sure everyone really appreciates it!

 

Best wishes

vicki

 

ESSAY 341

 

A 24 year old nulliparous woman has been referred to the gynaecology clinic because of problems with contraception. She has 3 unplanned pregnancies resulting in early medical termination. Her BMI is 42 kg/m2.

 (a) Discuss your clinical assessment [6 marks].

When assessing this patient in clinic I would initially start by taking a detailed gynaecological history especially assessing her last menstrual period dates, the length of menstrual cycles, pregnancy history, any concurrent menstrual problems such as dysmenorrhoea, menorrhagia, intermenstrual bleeding, dyspareunia, postcoital bleeding, symptoms of hyperandrogenaemia such as hirsuitism and acne. These questions are essential to rule out a potential ongoing pregnancy, any functional problems that either require further investigation prior to contraceptive initiation such as symptoms suggesting lower genital tract infections needing swabs or whether the contraceptive options could be tailored to ease these symptoms as a non contraceptive benefit.

A full sexual history including the last episode of unprotected sexual intercourse would be advisable to assess her risk of potential STIs and pelvic inflammatory disease which would preclude certain options of contraceptives. A contraceptive history would indicate what options have already been used, any side effects found with the method which may have affected her compliance and her ideas of what she would deem suitable or which options she would rule out straight away. I would also assess the events surrounding hte terminations of pregnancy especially whether they were contraception failure.

A detailed family history particularly of venous thromboembolic episodes in family members under the age of 45 (COCP UKMEC 3), or a strong family history of inherited cancers such as breast, or confirmed BRAC gene mutations (UKMEC 3 for COCP) which would rule out certain contraceptive options.

A past/ ongoing medical history for this patient is necessary to highlight any possible contraindications to certain contraceptives such as migraine with aura, history of DVT/PE, hypertension (although rare at 24 years old), hepatic disease (cirrhosis or liver tumours), or having treatment for any female specific cancers especially breast (again rare at this age).

I would take a full social history mainly highlighting on smoking history (as part of a cardiovascular risk factor work up including the known obesity risk) and stability of lifestyle to assess the ability to manage daily compliance of contraception or if long acting reversible contraception would be more favourable.

A drug history would need to include hepatic enzyme inducing drugs for epilepsy (or antibiotics), certain antiretroviral medications which despite being UKMEC 2 for most contraceptives would need ongoing barrier contraction as efficacy is substantially reduced. I would also elucidate if any past medical diseases had been missed out with ongoing medications such as anti-hypertensive’s.

As part of my examination work up, to commence most contraceptives a formal internal or breast examination is not necessary unless the history suggests an ongoing problem or an intrauterine coil is being considered when swabs would be also required. A blood pressure measurement is essential especially given her BMI. If there are concerns regarding a potential pregnancy I would perform a pregnancy test.

(b) Discuss the options for hormonal contraception [10 marks].

A few contraceptive options are ruled out for this patient as her BMI of greater than 35 which is UKMEC 3 (risk outweighs benefit). These include the combined oral contraceptive pill, patch and vaginal ring.

Progesterone only pills are very restricted regarding their timing of taking it every day (within 3 hours). Cerazette (desogestrel) has a 12 hour window. The benefit of taking a progesterone only pill is that is taken every day so compliance is increased compared with the combined pill. The typical failure rate for progesterone only pills 0.3-0.8%. There is no evidence that the POP may be less effective in women with a weight greater than 70kg. Side effects of the POP include irregular and potentially prolonged bleeding patterns as well as amenorrhoea. Other progestogenic side effects of bloating, acne, breast tenderness and mood swings need to be discussed with the patient. Missed pills with the traditional POP (MICRONOR, FEMULEN) will need extra protection for 48 hours but Cerazette will need a 7 day window of barrier protection of abstinence. Non contraceptive benefits include reduction in dysmenorrhoea. This option needs high motivation for daily compliance and would not be suitable for a patient likely to forget or in a patient who has had previous contraceptive failures and multiple terminations.

Progesterone only injectables including depo-provera and NET-EN are part of the Long acting reversible contraceptive group (LARC). They are not user dependent although the injection has to be repeated every 12-14 weeks. A typical user failure rate of the injection is <0.4/100 woman years. Obesity does not reduce the injection’s efficacy but a side effect of the injection is a weight gain of average 2-3kg in a year especially if a patient’s BMI is greater than 30. Other side effects include irregular and prolonged vaginal bleeding and amenorrhoea. There is an associated small loss in bone mineral density with the injection which may not be entirely reversible however this patient is aged 24 therefore peak bone mass will be reached. Fertility can be delayed up to a year for this form of contraception.

Progesterone only implants (IMPLANON) have a licensed use for 3 years however the efficacy in the third year may be reduced in obese patients. The etonorgestrel implant is inserted in the medial aspect of the upper arm therefore is not user dependent. A typical user rate for implanon is less than 0.1/100 woman years. Side effects include irregular or persistent bleeding which causes nearly 50% of women to stop using the implant.  Fertility of restored within the first month of implant removal. The implant does not require further follow up until it needs replacing.

(c) Discuss the use of intra-uterine contraception in this woman [4 marks].

Intrauterine copper contraceptive devices (IUCD) or the intrauterine system (IUS) are both feasible options for this patient as both are LARCs suitable for nulliparous women. The IUCD is licensed for use between 5-8 years with a failure rate of 0.2% if the coil contains over 380mm2 of copper. The expulsion rate is low (less than 5%) especially with Gynae fix. They are easier to insert in nulliparous women than the IUS and is effective immediately. Prudent to this patient, there is a 6 times increase risk of pelvic inflammatory disease in the first 3 weeks of insertion with both types of IUCD and IUS which would be reduced by prophylactic antibiotic cover or triple swabs taken pre-procedure. If a pregnancy occurs however there is a 1:20 chance that the pregnancy is an ectopic gestation and the woman should be counselled appropriately prior to consenting for both the IUCD and IUS. The side effects/ complication of the IUCD include menorrhagia and uterine perforation (less than 1% risk).

The IUS is licensed for 5 years with a failure rate of less than 0.1% with an expulsion rate of less than 5% in 5 years. Side effects include irregular prolonged light bleeding and amenorrhoea and it also carries a less than 1% risk of uterine perforation. It has an added benefit of reducing 97% of menstrual flow by the end of the first year.

Both of these options are user independent and have no altered efficacy with obesity. There is no delay to future fertility for both of these options.